Mathematical and Physical Journal
for High Schools
Issued by the MATFUND Foundation
Already signed up?
New to KöMaL?

Problem B. 5044. (September 2019)

B. 5044. Let \(\displaystyle D\) be a point in the interior of side \(\displaystyle AB\) in triangle \(\displaystyle ABC\), and \(\displaystyle E\) be a point in the interior of side \(\displaystyle AC\). The intersection of line segments \(\displaystyle BE\) and \(\displaystyle CD\) is \(\displaystyle M\). Let \(\displaystyle x\) denote the area of triangle \(\displaystyle BCM\), and let \(\displaystyle y\) denote the area of triangle \(\displaystyle EDM\). Prove that \(\displaystyle T_{ABC}\ge x \frac{\sqrt x+\sqrt y}{\sqrt x-\sqrt y}\).

(6 pont)

Deadline expired on October 10, 2019.


Sorry, the solution is available only in Hungarian. Google translation

1. megoldás.

Legyen \(\displaystyle p,q\in (0,1)\) úgy, hogy \(\displaystyle AD\colon DB=p\colon (1-p)\), \(\displaystyle AE\colon EC=q\colon (1-q)\), és az általánosság megszorítása nélkül tegyük fel, hogy \(\displaystyle T_{ABC}=1\). Írjuk fel az \(\displaystyle ADC\triangle\)-re és a \(\displaystyle BE\) egyenesre a Menelaosz-tételt (az irányítást figyelmen kívül hagyva):

\(\displaystyle \frac{AB}{BD}\cdot \frac{DM}{MC}\cdot \frac{CE}{EA}=1.\)

A bevezetett jelölések szerint \(\displaystyle AB \colon BD= 1\colon (1-p)\) és \(\displaystyle CE \colon EA=(1-q) \colon q\), ezekből pedig \(\displaystyle DM\colon MC=q(1-p) \colon (1-q)\) adódik, s ugyanígy kiszámítható, hogy \(\displaystyle EM\colon MA= p(1-q)\colon (1-p)\).

Vegyük észre, hogy a \(\displaystyle BCM\) és \(\displaystyle BCD\) háromszögek \(\displaystyle B\) csúcsból induló magassága közös, ezért \(\displaystyle T_{BCM}\colon T_{BCD}=CM \colon CD\); s hasonlóan a \(\displaystyle BCD\) és \(\displaystyle ABC\) háromszögek \(\displaystyle C\)-ből induló magassága közös, ezért \(\displaystyle T_{BCD}\colon T_{ABC}=BD \colon BA\). Felhasználva ezeket, a \(\displaystyle T_{ABC}=1\) feltevést, valamint a \(\displaystyle DM\colon MC=q(1-p) \colon (1-q)\) összefüggésből következő \(\displaystyle MC\colon DC = (1-q) \colon (1-pq)\) arányt kapjuk, hogy:

\(\displaystyle x=T_{BCD} \cdot \frac{MC}{DC}=T_{ABC}\cdot \frac{DB}{AB} \cdot \frac{MC}{DC}=(1-p)\cdot \frac{1-q}{1-pq}=\frac{(1-p)(1-q)}{1-pq}.\)

Másrészről bevezetve a \(\displaystyle \mu=DME\angle=CMB\angle\) jelölést:

\(\displaystyle \frac yx=\frac{2y}{2x}=\frac{ME\cdot MD \cdot \sin \mu}{MB \cdot MC \cdot \sin \mu }=\frac {DM}{MC}\cdot \frac{EM}{MB} =\frac{q(1-p)}{1-q}\cdot \frac{p(1-q)}{1-p}=pq.\)

A kapott összefüggéseket beírva, az igazolandó a következő alakot ölti:

\(\displaystyle 1\geq x \dfrac{\sqrt x+\sqrt y}{\sqrt x-\sqrt y}= x \cdot \frac{1+\sqrt{y/x}}{1-\sqrt{y/x}}=\frac{(1-p)(1-q)}{1-pq}\cdot \frac{1+\sqrt{pq}}{1-\sqrt{pq}},\)

ahol \(\displaystyle p,q\in (0,1).\) A kapott kifejezést egyszerűsítsük \(\displaystyle (1+\sqrt{pq})\)-val, majd alkalmazzuk a \(\displaystyle p+q\geq 2\sqrt{pq}\) számtani-mértani közép egyenlőtlenséget:

\(\displaystyle \frac{(1-p)(1-q)}{1-pq}\cdot \frac{1+\sqrt{pq}}{1-\sqrt{pq}}=\frac{(1-p)(1-q)}{(1-\sqrt{pq})^2}=\frac{1-p-q+pq}{1-2\sqrt{pq}+pq}\le \frac{1-2\sqrt{pq}+pq}{1-2\sqrt{pq}+pq}= 1.\)

Ezzel az állítást beláttuk. Egyenlőség pontosan akkor van, amikor a használt számtani-mértani közép egyenlőtlenségben is, azaz \(\displaystyle p=q\) esetén. A párhuzamos szelők tétele szerint ez pontosan \(\displaystyle DE\parallel BC\) esetén teljesül.

2. megoldás. Legyen \(\displaystyle P=AM\cap BC\), \(\displaystyle Q=AM\cap DE\), \(\displaystyle t=T_{ABC}\), \(\displaystyle b=T_{BMD}\) és \(\displaystyle c=T_{CME}\).

Az \(\displaystyle (AMPQ)\) négyes harmonikus, ezért

\(\displaystyle 1 = \frac{AQ}{MQ}\cdot\frac{MP}{AP} = \frac{T_{DEA}}{T_{DEM}}\cdot\frac{T_{BCM}}{T_{BCA}} = \frac{t-x-y-b-c}{y}\cdot\frac{x}{t}; \)

rendezve

\(\displaystyle (x-y) t = x(x+y+b+c). \)

(Az is látszik, hogy \(\displaystyle x>y\).)

A \(\displaystyle BCED\) négyszögben a szemközti háromszögek területének szorzata egyenlő, \(\displaystyle bc=xy\); a számtani és mértani közepek közötti egyenlőtlenségből \(\displaystyle b+c\ge2\sqrt{bc}=2\sqrt{xy}\), ezért

\(\displaystyle t = \frac{x(x+y+b+c)}{x-y} \ge \frac{x(x+y+2\sqrt{xy})}{x-y} = \frac{x\big(\sqrt{x}+\sqrt{y}\big)^2}{\big(\sqrt{x}+\sqrt{y}\big)\big(\sqrt{x}-\sqrt{y}\big)} = \frac{x\big(\sqrt{x}+\sqrt{y}\big)}{\sqrt{x}-\sqrt{y}}. \)

Akkor van egyenlőség, ha \(\displaystyle b=c\), vagyis ha \(\displaystyle DE\big\|BC\).


Statistics:

21 students sent a solution.
6 points:Andó Viola, Bán-Szabó Áron, Beke Csongor, Czett Mátyás, Füredi Erik Benjámin, Geretovszky Anna, Hámori Janka, Jánosik Áron, Kerekes Boldizsár, Lengyel Ádám, Molnár Lehel, Nguyen Bich Diep, Rareș Polenciuc, Somogyi Dalma, Tiderenczl Dániel, Vágó Bendegúz Zsolt.
5 points:Hegedűs Dániel.
3 points:1 student.
2 points:1 student.
1 point:1 student.
0 point:1 student.

Problems in Mathematics of KöMaL, September 2019